Cannon A --> RA contracts against closed tricuspid valve Prominent v wave--->tricuspid regurgitation Kussmaul's--> pericarditis,restrictive cardiomyopathy...
1 month ago | 3
Cannon waves are seen in tricuspid stenosis in which case diastolic nurmur is heard.
1 month ago | 0
Kindly Comment on the clinical conditions in which cannon A waves, prominent V waves, and Kussmaul’s sign are observed!
1 month ago | 0
Intellect Medicos
A 45-year-old man presents with exertional breathlessness and fatigue over several months. On examination, there is a systolic ejection murmur heard best at the upper left sternal border.
Jugular venous pressure is assessed carefully. There is no peripheral oedema, and blood pressure is within normal limits. Heart sounds are normal apart from the murmur.
Which of the following JVP findings is most likely in this patient?
1 month ago | [YT] | 161